Is there a simple example that empirical evidence is misleading?How do I become a Scarer?How to nurture a good student?Dyscalculia and studying mathematics (as major)What to do if there is a disagreement on fundamentals, e.g. axioms or inference rules?Metonymy in mathematicsHow to prove Taylor formulas?Logic in symbols or wordsEffectiveness of students seeing proofs - reference requestIs there any research on the value of extra credit in the college mathematics classroom?Inability to work with an arbitrary mathematical object

Cisco 3750X Power Cable

Why did Drogon spare this character?

Is it safe to redirect stdout and stderr to the same file without file descriptor copies?

Merge pdfs sequentially

Writing "hahaha" versus describing the laugh

Time complexity of an algorithm: Is it important to state the base of the logarithm?

Why is the Eisenstein ideal paper so great?

Count all vowels in string

Testing using real data of the customer

What could be my risk mitigation strategies if my client wants to contract UAT?

How can I minimize the damage of an unstable nuclear reactor to the surrounding area?

What is the limit to a Glyph of Warding's trigger?

To exponential digit growth and beyond!

What is the purpose of the yellow wired panels on the IBM 360 Model 20?

What did the 'turbo' button actually do?

Where is Jon going?

Why A=2 and B=1 in the call signs for Spirit and Opportunity?

Team has team lunch everyday, am I forced to go?

Is this homebrew "Cactus Grenade" cantrip balanced?

Split into three!

Possibility of faking someone's public key

Ribbon Cable Cross Talk - Is there a fix after the fact?

Are there historical examples of audiences drawn to a work that was "so bad it's good"?

How does Dreadhorde Arcanist interact with split cards?



Is there a simple example that empirical evidence is misleading?


How do I become a Scarer?How to nurture a good student?Dyscalculia and studying mathematics (as major)What to do if there is a disagreement on fundamentals, e.g. axioms or inference rules?Metonymy in mathematicsHow to prove Taylor formulas?Logic in symbols or wordsEffectiveness of students seeing proofs - reference requestIs there any research on the value of extra credit in the college mathematics classroom?Inability to work with an arbitrary mathematical object













3












$begingroup$


Suppose that I want to show a student that emperical evidence in mathematics is not enough and we do need proofs, what kind of examples can I use?



By emperical evidence, I mean (most of the time) you cannot simply check the statement $S(n)$ for $n in 1,dots, 10^9$ and conclude it's true for all $n in mathbb N$.










share|improve this question











$endgroup$







  • 2




    $begingroup$
    math.stackexchange.com/questions/514/…
    $endgroup$
    – Jasper
    8 hours ago






  • 1




    $begingroup$
    mathoverflow.net/q/15444/36173
    $endgroup$
    – Paracosmiste
    6 hours ago










  • $begingroup$
    If you pick $n$ (generic) points on a circle and connect them with lines, the disc divides into a number of regions. It appears to be a power of two for $nleq 5$, then it changes. See the nice article: quantamagazine.org/…
    $endgroup$
    – Adam
    2 hours ago















3












$begingroup$


Suppose that I want to show a student that emperical evidence in mathematics is not enough and we do need proofs, what kind of examples can I use?



By emperical evidence, I mean (most of the time) you cannot simply check the statement $S(n)$ for $n in 1,dots, 10^9$ and conclude it's true for all $n in mathbb N$.










share|improve this question











$endgroup$







  • 2




    $begingroup$
    math.stackexchange.com/questions/514/…
    $endgroup$
    – Jasper
    8 hours ago






  • 1




    $begingroup$
    mathoverflow.net/q/15444/36173
    $endgroup$
    – Paracosmiste
    6 hours ago










  • $begingroup$
    If you pick $n$ (generic) points on a circle and connect them with lines, the disc divides into a number of regions. It appears to be a power of two for $nleq 5$, then it changes. See the nice article: quantamagazine.org/…
    $endgroup$
    – Adam
    2 hours ago













3












3








3





$begingroup$


Suppose that I want to show a student that emperical evidence in mathematics is not enough and we do need proofs, what kind of examples can I use?



By emperical evidence, I mean (most of the time) you cannot simply check the statement $S(n)$ for $n in 1,dots, 10^9$ and conclude it's true for all $n in mathbb N$.










share|improve this question











$endgroup$




Suppose that I want to show a student that emperical evidence in mathematics is not enough and we do need proofs, what kind of examples can I use?



By emperical evidence, I mean (most of the time) you cannot simply check the statement $S(n)$ for $n in 1,dots, 10^9$ and conclude it's true for all $n in mathbb N$.







undergraduate-education






share|improve this question















share|improve this question













share|improve this question




share|improve this question








edited 3 hours ago









Rusty Core

18319




18319










asked 9 hours ago









ablmfablmf

24219




24219







  • 2




    $begingroup$
    math.stackexchange.com/questions/514/…
    $endgroup$
    – Jasper
    8 hours ago






  • 1




    $begingroup$
    mathoverflow.net/q/15444/36173
    $endgroup$
    – Paracosmiste
    6 hours ago










  • $begingroup$
    If you pick $n$ (generic) points on a circle and connect them with lines, the disc divides into a number of regions. It appears to be a power of two for $nleq 5$, then it changes. See the nice article: quantamagazine.org/…
    $endgroup$
    – Adam
    2 hours ago












  • 2




    $begingroup$
    math.stackexchange.com/questions/514/…
    $endgroup$
    – Jasper
    8 hours ago






  • 1




    $begingroup$
    mathoverflow.net/q/15444/36173
    $endgroup$
    – Paracosmiste
    6 hours ago










  • $begingroup$
    If you pick $n$ (generic) points on a circle and connect them with lines, the disc divides into a number of regions. It appears to be a power of two for $nleq 5$, then it changes. See the nice article: quantamagazine.org/…
    $endgroup$
    – Adam
    2 hours ago







2




2




$begingroup$
math.stackexchange.com/questions/514/…
$endgroup$
– Jasper
8 hours ago




$begingroup$
math.stackexchange.com/questions/514/…
$endgroup$
– Jasper
8 hours ago




1




1




$begingroup$
mathoverflow.net/q/15444/36173
$endgroup$
– Paracosmiste
6 hours ago




$begingroup$
mathoverflow.net/q/15444/36173
$endgroup$
– Paracosmiste
6 hours ago












$begingroup$
If you pick $n$ (generic) points on a circle and connect them with lines, the disc divides into a number of regions. It appears to be a power of two for $nleq 5$, then it changes. See the nice article: quantamagazine.org/…
$endgroup$
– Adam
2 hours ago




$begingroup$
If you pick $n$ (generic) points on a circle and connect them with lines, the disc divides into a number of regions. It appears to be a power of two for $nleq 5$, then it changes. See the nice article: quantamagazine.org/…
$endgroup$
– Adam
2 hours ago










2 Answers
2






active

oldest

votes


















3












$begingroup$

There are some collections of such examples at sister sites:



  • Conjectures that have been disproved with extremely large counterexamples?
    at Mathematics Stack Exchange.


  • Examples of eventual counterexamples at MathOverflow.



One rather simple example that can be checked with a calculator is the conjecture by Fermat, that all numbers of the form $$2^2^n+1, qquad n in mathbb N_0$$ are prime.



In fact,




  • $2^2^0 +1 = 3$ is prime


  • $2^2^1 +1 = 5$ is prime


  • $2^2^2 +1 = 17$ is prime


  • $2^2^3 +1 = 257$ is prime


  • $2^2^4 +1 = 65537$ is prime


  • $2^2^5 +1 = 4294967297$ is not prime: $4294967297 = 641 cdot 6700417$

So the original conjecture is clearly false, but it took nearly 100 years to find the counterexample. All following Fermat numbers appear to be composite, but this is an open problem.






share|improve this answer









$endgroup$




















    3












    $begingroup$

    Strangely, just this morning I asked Wolfram|Alpha to compute the sum $$sum_n=1^inftyfrac1nsin(n)$$ and it returned the approximate value of $-0.863507$. I asked it to "show more digits", and it returned a new approximation:



    $94.377284731050845020943145217217734512865979242824685504875914407196948018$



    I was trying to illustrate a series whose convergence (or divergence) is difficult to determine and was treated to some very different approximations. Note that Wolfram did not tell whether the series converges.






    share|improve this answer











    $endgroup$












    • $begingroup$
      FWIW: You can see in MSE 665776 that the sequence $frac1n sin(n)$ does not converge; in particular, the sequence does not converge to $0$. So, the series described here does not converge.
      $endgroup$
      – Benjamin Dickman
      26 mins ago











    Your Answer








    StackExchange.ready(function()
    var channelOptions =
    tags: "".split(" "),
    id: "548"
    ;
    initTagRenderer("".split(" "), "".split(" "), channelOptions);

    StackExchange.using("externalEditor", function()
    // Have to fire editor after snippets, if snippets enabled
    if (StackExchange.settings.snippets.snippetsEnabled)
    StackExchange.using("snippets", function()
    createEditor();
    );

    else
    createEditor();

    );

    function createEditor()
    StackExchange.prepareEditor(
    heartbeatType: 'answer',
    autoActivateHeartbeat: false,
    convertImagesToLinks: false,
    noModals: true,
    showLowRepImageUploadWarning: true,
    reputationToPostImages: null,
    bindNavPrevention: true,
    postfix: "",
    imageUploader:
    brandingHtml: "Powered by u003ca class="icon-imgur-white" href="https://imgur.com/"u003eu003c/au003e",
    contentPolicyHtml: "User contributions licensed under u003ca href="https://creativecommons.org/licenses/by-sa/3.0/"u003ecc by-sa 3.0 with attribution requiredu003c/au003e u003ca href="https://stackoverflow.com/legal/content-policy"u003e(content policy)u003c/au003e",
    allowUrls: true
    ,
    noCode: true, onDemand: true,
    discardSelector: ".discard-answer"
    ,immediatelyShowMarkdownHelp:true
    );



    );













    draft saved

    draft discarded


















    StackExchange.ready(
    function ()
    StackExchange.openid.initPostLogin('.new-post-login', 'https%3a%2f%2fmatheducators.stackexchange.com%2fquestions%2f16636%2fis-there-a-simple-example-that-empirical-evidence-is-misleading%23new-answer', 'question_page');

    );

    Post as a guest















    Required, but never shown

























    2 Answers
    2






    active

    oldest

    votes








    2 Answers
    2






    active

    oldest

    votes









    active

    oldest

    votes






    active

    oldest

    votes









    3












    $begingroup$

    There are some collections of such examples at sister sites:



    • Conjectures that have been disproved with extremely large counterexamples?
      at Mathematics Stack Exchange.


    • Examples of eventual counterexamples at MathOverflow.



    One rather simple example that can be checked with a calculator is the conjecture by Fermat, that all numbers of the form $$2^2^n+1, qquad n in mathbb N_0$$ are prime.



    In fact,




    • $2^2^0 +1 = 3$ is prime


    • $2^2^1 +1 = 5$ is prime


    • $2^2^2 +1 = 17$ is prime


    • $2^2^3 +1 = 257$ is prime


    • $2^2^4 +1 = 65537$ is prime


    • $2^2^5 +1 = 4294967297$ is not prime: $4294967297 = 641 cdot 6700417$

    So the original conjecture is clearly false, but it took nearly 100 years to find the counterexample. All following Fermat numbers appear to be composite, but this is an open problem.






    share|improve this answer









    $endgroup$

















      3












      $begingroup$

      There are some collections of such examples at sister sites:



      • Conjectures that have been disproved with extremely large counterexamples?
        at Mathematics Stack Exchange.


      • Examples of eventual counterexamples at MathOverflow.



      One rather simple example that can be checked with a calculator is the conjecture by Fermat, that all numbers of the form $$2^2^n+1, qquad n in mathbb N_0$$ are prime.



      In fact,




      • $2^2^0 +1 = 3$ is prime


      • $2^2^1 +1 = 5$ is prime


      • $2^2^2 +1 = 17$ is prime


      • $2^2^3 +1 = 257$ is prime


      • $2^2^4 +1 = 65537$ is prime


      • $2^2^5 +1 = 4294967297$ is not prime: $4294967297 = 641 cdot 6700417$

      So the original conjecture is clearly false, but it took nearly 100 years to find the counterexample. All following Fermat numbers appear to be composite, but this is an open problem.






      share|improve this answer









      $endgroup$















        3












        3








        3





        $begingroup$

        There are some collections of such examples at sister sites:



        • Conjectures that have been disproved with extremely large counterexamples?
          at Mathematics Stack Exchange.


        • Examples of eventual counterexamples at MathOverflow.



        One rather simple example that can be checked with a calculator is the conjecture by Fermat, that all numbers of the form $$2^2^n+1, qquad n in mathbb N_0$$ are prime.



        In fact,




        • $2^2^0 +1 = 3$ is prime


        • $2^2^1 +1 = 5$ is prime


        • $2^2^2 +1 = 17$ is prime


        • $2^2^3 +1 = 257$ is prime


        • $2^2^4 +1 = 65537$ is prime


        • $2^2^5 +1 = 4294967297$ is not prime: $4294967297 = 641 cdot 6700417$

        So the original conjecture is clearly false, but it took nearly 100 years to find the counterexample. All following Fermat numbers appear to be composite, but this is an open problem.






        share|improve this answer









        $endgroup$



        There are some collections of such examples at sister sites:



        • Conjectures that have been disproved with extremely large counterexamples?
          at Mathematics Stack Exchange.


        • Examples of eventual counterexamples at MathOverflow.



        One rather simple example that can be checked with a calculator is the conjecture by Fermat, that all numbers of the form $$2^2^n+1, qquad n in mathbb N_0$$ are prime.



        In fact,




        • $2^2^0 +1 = 3$ is prime


        • $2^2^1 +1 = 5$ is prime


        • $2^2^2 +1 = 17$ is prime


        • $2^2^3 +1 = 257$ is prime


        • $2^2^4 +1 = 65537$ is prime


        • $2^2^5 +1 = 4294967297$ is not prime: $4294967297 = 641 cdot 6700417$

        So the original conjecture is clearly false, but it took nearly 100 years to find the counterexample. All following Fermat numbers appear to be composite, but this is an open problem.







        share|improve this answer












        share|improve this answer



        share|improve this answer










        answered 5 hours ago









        JasperJasper

        799513




        799513





















            3












            $begingroup$

            Strangely, just this morning I asked Wolfram|Alpha to compute the sum $$sum_n=1^inftyfrac1nsin(n)$$ and it returned the approximate value of $-0.863507$. I asked it to "show more digits", and it returned a new approximation:



            $94.377284731050845020943145217217734512865979242824685504875914407196948018$



            I was trying to illustrate a series whose convergence (or divergence) is difficult to determine and was treated to some very different approximations. Note that Wolfram did not tell whether the series converges.






            share|improve this answer











            $endgroup$












            • $begingroup$
              FWIW: You can see in MSE 665776 that the sequence $frac1n sin(n)$ does not converge; in particular, the sequence does not converge to $0$. So, the series described here does not converge.
              $endgroup$
              – Benjamin Dickman
              26 mins ago















            3












            $begingroup$

            Strangely, just this morning I asked Wolfram|Alpha to compute the sum $$sum_n=1^inftyfrac1nsin(n)$$ and it returned the approximate value of $-0.863507$. I asked it to "show more digits", and it returned a new approximation:



            $94.377284731050845020943145217217734512865979242824685504875914407196948018$



            I was trying to illustrate a series whose convergence (or divergence) is difficult to determine and was treated to some very different approximations. Note that Wolfram did not tell whether the series converges.






            share|improve this answer











            $endgroup$












            • $begingroup$
              FWIW: You can see in MSE 665776 that the sequence $frac1n sin(n)$ does not converge; in particular, the sequence does not converge to $0$. So, the series described here does not converge.
              $endgroup$
              – Benjamin Dickman
              26 mins ago













            3












            3








            3





            $begingroup$

            Strangely, just this morning I asked Wolfram|Alpha to compute the sum $$sum_n=1^inftyfrac1nsin(n)$$ and it returned the approximate value of $-0.863507$. I asked it to "show more digits", and it returned a new approximation:



            $94.377284731050845020943145217217734512865979242824685504875914407196948018$



            I was trying to illustrate a series whose convergence (or divergence) is difficult to determine and was treated to some very different approximations. Note that Wolfram did not tell whether the series converges.






            share|improve this answer











            $endgroup$



            Strangely, just this morning I asked Wolfram|Alpha to compute the sum $$sum_n=1^inftyfrac1nsin(n)$$ and it returned the approximate value of $-0.863507$. I asked it to "show more digits", and it returned a new approximation:



            $94.377284731050845020943145217217734512865979242824685504875914407196948018$



            I was trying to illustrate a series whose convergence (or divergence) is difficult to determine and was treated to some very different approximations. Note that Wolfram did not tell whether the series converges.







            share|improve this answer














            share|improve this answer



            share|improve this answer








            edited 4 hours ago

























            answered 8 hours ago









            Nick CNick C

            2,260626




            2,260626











            • $begingroup$
              FWIW: You can see in MSE 665776 that the sequence $frac1n sin(n)$ does not converge; in particular, the sequence does not converge to $0$. So, the series described here does not converge.
              $endgroup$
              – Benjamin Dickman
              26 mins ago
















            • $begingroup$
              FWIW: You can see in MSE 665776 that the sequence $frac1n sin(n)$ does not converge; in particular, the sequence does not converge to $0$. So, the series described here does not converge.
              $endgroup$
              – Benjamin Dickman
              26 mins ago















            $begingroup$
            FWIW: You can see in MSE 665776 that the sequence $frac1n sin(n)$ does not converge; in particular, the sequence does not converge to $0$. So, the series described here does not converge.
            $endgroup$
            – Benjamin Dickman
            26 mins ago




            $begingroup$
            FWIW: You can see in MSE 665776 that the sequence $frac1n sin(n)$ does not converge; in particular, the sequence does not converge to $0$. So, the series described here does not converge.
            $endgroup$
            – Benjamin Dickman
            26 mins ago

















            draft saved

            draft discarded
















































            Thanks for contributing an answer to Mathematics Educators Stack Exchange!


            • Please be sure to answer the question. Provide details and share your research!

            But avoid


            • Asking for help, clarification, or responding to other answers.

            • Making statements based on opinion; back them up with references or personal experience.

            Use MathJax to format equations. MathJax reference.


            To learn more, see our tips on writing great answers.




            draft saved


            draft discarded














            StackExchange.ready(
            function ()
            StackExchange.openid.initPostLogin('.new-post-login', 'https%3a%2f%2fmatheducators.stackexchange.com%2fquestions%2f16636%2fis-there-a-simple-example-that-empirical-evidence-is-misleading%23new-answer', 'question_page');

            );

            Post as a guest















            Required, but never shown





















































            Required, but never shown














            Required, but never shown












            Required, but never shown







            Required, but never shown

































            Required, but never shown














            Required, but never shown












            Required, but never shown







            Required, but never shown







            Popular posts from this blog

            Log på Navigationsmenu

            Wonderful Copenhagen (sang) Eksterne henvisninger | NavigationsmenurSide på frankloesser.comWonderful Copenhagen

            Detroit Tigers Spis treści Historia | Skład zespołu | Sukcesy | Członkowie Baseball Hall of Fame | Zastrzeżone numery | Przypisy | Menu nawigacyjneEncyclopedia of Detroit - Detroit TigersTigers Stadium, Detroit, MITigers Timeline 1900sDetroit Tigers Team History & EncyclopediaTigers Timeline 1910s1935 World Series1945 World Series1945 World Series1984 World SeriesComerica Park, Detroit, MI2006 World Series2012 World SeriesDetroit Tigers 40-Man RosterDetroit Tigers Coaching StaffTigers Hall of FamersTigers Retired Numberse